Difference between revisions of "2015 AMC 8 Problems/Problem 25"

(See Also)
(Video Solutions)
 
(59 intermediate revisions by 30 users not shown)
Line 1: Line 1:
 +
== Problem ==
 +
 +
 
One-inch squares are cut from the corners of this 5 inch square.  What is the area in square inches of the largest square that can be fitted into the remaining space?
 
One-inch squares are cut from the corners of this 5 inch square.  What is the area in square inches of the largest square that can be fitted into the remaining space?
 
<math>\textbf{(A)  } 9\qquad \textbf{(B)  } 12\frac{1}{2}\qquad \textbf{(C)  } 15\qquad \textbf{(D)  } 15\frac{1}{2}\qquad \textbf{(E)  } 17</math>
 
  
 
<asy>
 
<asy>
 +
size(75);
 
draw((0,0)--(0,5)--(5,5)--(5,0)--cycle);
 
draw((0,0)--(0,5)--(5,5)--(5,0)--cycle);
 
filldraw((0,4)--(1,4)--(1,5)--(0,5)--cycle, gray);
 
filldraw((0,4)--(1,4)--(1,5)--(0,5)--cycle, gray);
Line 11: Line 13:
 
</asy>
 
</asy>
  
===Solution 1===
+
<math>\textbf{(A) } 9\qquad \textbf{(B} 12\frac{1}{2}\qquad \textbf{(C)  } 15\qquad \textbf{(D) } 15\frac{1}{2}\qquad \textbf{(E)  } 17</math>
We can draw a diagram as shown.
 
<asy>
 
draw((0,0)--(0,5)--(5,5)--(5,0)--cycle);
 
filldraw((0,4)--(1,4)--(1,5)--(0,5)--cycle, gray);
 
filldraw((0,0)--(1,0)--(1,1)--(0,1)--cycle, gray);
 
filldraw((4,0)--(4,1)--(5,1)--(5,0)--cycle, gray);
 
filldraw((4,4)--(4,5)--(5,5)--(5,4)--cycle, gray);
 
path arc = arc((2.5,4),1.5,0,90);
 
pair P = intersectionpoint(arc,(0,5)--(5,5));
 
pair Pp=rotate(90,(2.5,2.5))*P, Ppp = rotate(90,(2.5,2.5))*Pp, Pppp=rotate(90,(2.5,2.5))*Ppp;
 
draw(P--Pp--Ppp--Pppp--cycle);
 
</asy>
 
Let us focus on the big triangles taking up the rest of the space. The triangles on top of the unit square between the inscribed square, are similiar to the <math>4</math> big triangles by <math>AA.</math>  Let the height of a big triangle be <math>x</math> then <math>\tfrac{x}{x-1}=\tfrac{5-x}{1}</math>.
 
<cmath>x=-x^2+6x-5</cmath>
 
<cmath>x^2-5x+5=0</cmath>
 
<cmath>x=\dfrac{5\pm \sqrt{(-5)^2-(4)(1)(5)}}{2}</cmath>
 
<cmath>x=\dfrac{5\pm \sqrt{5}}{2}</cmath>
 
Thus <math>x=\dfrac{5-\sqrt{5}}{2}</math>, because by symmetry, <math>x < \dfrac52</math>.
 
  
This means the area of each triangle is <math>\dfrac{5-\sqrt{5}}{2}*(5-\dfrac{5-\sqrt{5}}{2})*\dfrac{1}{2}=\dfrac{5}{2}</math>
 
Thus the area of the square is <math>25-(4*\dfrac{5}{2})=\boxed{\textbf{(C)}~15}</math>
 
  
===Solution 2=== 
+
==Video Solutions==
  
We draw a square as shown:
 
  
<asy>
 
draw((0,0)--(0,5)--(5,5)--(5,0)--cycle);
 
filldraw((0,4)--(1,4)--(1,5)--(0,5)--cycle, gray);
 
filldraw((0,0)--(1,0)--(1,1)--(0,1)--cycle, gray);
 
filldraw((4,0)--(4,1)--(5,1)--(5,0)--cycle, gray);
 
filldraw((4,4)--(4,5)--(5,5)--(5,4)--cycle, gray);
 
path arc = arc((2.5,4),1.5,0,90);
 
pair P = intersectionpoint(arc,(0,5)--(5,5));
 
pair Pp=rotate(90,(2.5,2.5))*P, Ppp = rotate(90,(2.5,2.5))*Pp, Pppp=rotate(90,(2.5,2.5))*Ppp;
 
draw(P--Pp--Ppp--Pppp--cycle);
 
filldraw((1,4)--P--(4,4)--cycle,red);
 
filldraw((4,4)--Pppp--(4,1)--cycle,red);
 
filldraw((1,1)--Ppp--(4,1)--cycle,red);
 
filldraw((1,1)--Pp--(1,4)--cycle,red);
 
</asy>
 
  
  
We wish to find the area of the square. The area of the larger square is composed of the smaller square and the four red triangles. The red triangles have base <math>3</math> and height <math>1</math>, so the combined area of the four triangles is <math>4 \cdot \frac 32=6</math>. The area of the smaller square is <math>9</math>. We add these to see that the area of the large square is <math>9+6=\boxed{{\textbf{(C)}}~15}</math>.
+
https://youtu.be/rTljQV79PCY
  
===Solution 3===
+
~savannahsolver
  
 +
https://youtu.be/51K3uCzntWs?t=3358
  
Let us find the area of the triangles and the unit squares: on each side, there are two triangles. They both have one leg of length <math>1</math>, and let's label the other legs <math>x</math> for one of the triangles and <math>y</math> for the other. Note that <math>x + y = 3</math>.
+
~ pi_is_3.14
The area of each of the triangles is <math>\frac{x}{2}</math> and <math>\frac{y}{2}</math>, and there are <math>4</math> of each. So now we need to find <math>4\left(\frac{x}{2}\right) + 4\left(\frac{y}{2}\right)</math>.
 
  
<math>(4)\frac{x}{2} + (4)\frac{y}{2}</math>
+
==See Also==
<math>\Rightarrow~~4\left(\frac{x}{2}+ \frac{y}{2}\right)</math>
 
<math>\Rightarrow~~4\left(\frac{x+y}{2}\right)</math>
 
Remember that <math>x+y=3</math>, so substituting this in we find that the area of all of the triangles is <math>4\left(\frac{3}{2}\right) = 6</math>.
 
The area of the <math>4</math> unit squares is <math>4</math>, so the area of the square we need is <math>25- (4+6) = \boxed{\textbf{(C)}~15}</math>
 
  
==See Also==
 
  
 
{{AMC8 box|year=2015|num-b=24|after=Last Problem}}
 
{{AMC8 box|year=2015|num-b=24|after=Last Problem}}
 
{{MAA Notice}}
 
{{MAA Notice}}

Latest revision as of 10:02, 23 July 2024

Problem

One-inch squares are cut from the corners of this 5 inch square. What is the area in square inches of the largest square that can be fitted into the remaining space?

[asy] size(75); draw((0,0)--(0,5)--(5,5)--(5,0)--cycle); filldraw((0,4)--(1,4)--(1,5)--(0,5)--cycle, gray); filldraw((0,0)--(1,0)--(1,1)--(0,1)--cycle, gray); filldraw((4,0)--(4,1)--(5,1)--(5,0)--cycle, gray); filldraw((4,4)--(4,5)--(5,5)--(5,4)--cycle, gray); [/asy]

$\textbf{(A)  } 9\qquad \textbf{(B)  } 12\frac{1}{2}\qquad \textbf{(C)  } 15\qquad \textbf{(D)  } 15\frac{1}{2}\qquad \textbf{(E)  } 17$


Video Solutions

https://youtu.be/rTljQV79PCY

~savannahsolver

https://youtu.be/51K3uCzntWs?t=3358

~ pi_is_3.14

See Also

2015 AMC 8 (ProblemsAnswer KeyResources)
Preceded by
Problem 24
Followed by
Last Problem
1 2 3 4 5 6 7 8 9 10 11 12 13 14 15 16 17 18 19 20 21 22 23 24 25
All AJHSME/AMC 8 Problems and Solutions

The problems on this page are copyrighted by the Mathematical Association of America's American Mathematics Competitions. AMC logo.png